Sard theorem in dimension one.

  • Thread starter Thread starter boboYO
  • Start date Start date
  • Tags Tags
    Dimension Theorem
Click For Summary
SUMMARY

The discussion focuses on proving the Sard theorem in one dimension, specifically that the set of critical values of a continuously differentiable function \( f: [a,b] \to \mathbb{R} \) has measure zero. The proof involves defining the set of critical points \( \text{Crit}(f) \) and demonstrating that the image of this set under \( f \) also has measure zero. The participants discuss the necessity of the continuity of the derivative and explore alternative approaches to the proof, ultimately confirming the validity of the argument presented.

PREREQUISITES
  • Understanding of the Sard theorem in real analysis
  • Knowledge of measure theory and concepts of measure zero
  • Familiarity with continuously differentiable functions
  • Application of the Mean Value Theorem (MVT)
NEXT STEPS
  • Study the proof of the Sard theorem in higher dimensions
  • Explore measure theory, focusing on sets of measure zero
  • Learn about the implications of the Mean Value Theorem in real analysis
  • Investigate the role of continuity in differentiable functions
USEFUL FOR

Mathematicians, students of real analysis, and anyone interested in advanced calculus and the properties of differentiable functions.

boboYO
Messages
106
Reaction score
0
Hi, I need some hints for the proof of the sard theorem in 1 dimension:

Prove that the set of critical values (f(x) where f'(x)=0) of continuously differentiable f:[a,b]->R has measure 0.My attempt:Fix \varepsilon, Let Crit(f) be the set of critical points of f. We want to show that f(Crit(f)) has measure 0.
Let C_k be the set of all x in Crit(f) such that |x-y|<\frac{1}{k} \implies |fx-fy|<\varepsilon|x-y|.

Clearly \bigcup_{k\in\mathbb{N}} C_k \supset \mathrm{Crit}(f), so it suffices to show that f\left(\bigcup_{k\in\mathbb{N}} C_k \right) has measure 0.

For any C_k, evenly split [a,b] into intervals I_1,\dots,I_n such that \frac{b-a}{n}<\frac{1}{k}.For each I_i, if there is a point of C_k in it, say x, then

|fx-fy|<\varepsilon|x-y| for all y in I_i.

Thus if I_i contains a point of C_k, then f(I_i) is contained in a open interval of at most 2w \varepsilon where w is the width of the interval.

Thus f(C_k) is contained in an open set of length at most 2w\varepsilon n =2(b-a) \varepsilon=K\varepsilon.

Since each f(C_k) has measure bounded above by K \varepsilon, and C_{k}\subset C_{k+1} for all k, then

\bigcup_{k\in\mathbb{N}} f(C_k) = f\left(\bigcup_{k\in\mathbb{N}} C_k \right) is also bounded above by K\varepsilon. Since \varepsilon was arbitrary we are done.
--

I am not completely convinced because I did not use the hypothesis that the derivative is continuous. Did I make a mistake? Is there a simpler way do this?
 
Last edited:
Physics news on Phys.org
I'm really having a hard time following that. It's not even clear to me that you even used that f is differentiable. Why don't you define C(epsilon) to be the set of all x such that f'(x)<epsilon. Then Crit(f) is contained in C(epsilon). Can you show the measure of f(C(epsilon)) is less than or equal to epsilon*measure(C(epsilon))?
 
(-\epsilon,\epsilon) is open and f'(x) is continuous so C_\epsilon is open too. An open set is a (at most) countable union of disjoint open intervals:

C_\epsilon=\bigcup_{i=1}^{\infty} I_i

For every I_i, if x and y are 2 points in I_i,

|fx-fy|&lt;\epsilon|x-y|&lt; \epsilon\mu(I_i), due to MVT. So \mu(f(I_i))&lt;\epsilon\mu(I_i)

Taking the union over i yields measure(f(Cε)) <ε*measure(Cε)<=ε(b-a) and we are done.

Thanks for your help :)
 
Last edited:
Now that I understand. And it even looks right.
 
Question: A clock's minute hand has length 4 and its hour hand has length 3. What is the distance between the tips at the moment when it is increasing most rapidly?(Putnam Exam Question) Answer: Making assumption that both the hands moves at constant angular velocities, the answer is ## \sqrt{7} .## But don't you think this assumption is somewhat doubtful and wrong?

Similar threads

  • · Replies 12 ·
Replies
12
Views
2K
  • · Replies 1 ·
Replies
1
Views
2K
  • · Replies 10 ·
Replies
10
Views
3K
  • · Replies 9 ·
Replies
9
Views
9K
  • · Replies 3 ·
Replies
3
Views
2K
  • · Replies 3 ·
Replies
3
Views
2K
Replies
18
Views
3K
Replies
4
Views
2K
  • · Replies 1 ·
Replies
1
Views
1K
  • · Replies 1 ·
Replies
1
Views
1K